You are on page 1of 6

Preptests 61 Answers and Explanations (By Ivy Global)

Section 3 Logical Reasoning


Questions 1 5 We cant really make many deductions in this game, so it really just involves keeping track of the rules and how they interact with each other. Its always good to keep numbers in mind in open distribution games like this, so we should remember that the split between cars can either be 3-3 or 2-4. 1. Type of Question: GlobalList This question should be approached with a rule sweep. a) b) c) d) e) This is the correct answer. Eliminated by rule #2: F or K must drive with J. Eliminated by rule #3: G and L must be together. Eliminated by rule #1: F or G must drive with H. Eliminated by rule #2: F or K must drive with J.

2. Type of Question: GlobalCannot Be True This question can be approached with a rule sweep. Any choice that doesnt have F or G as a driver is out (based on rule #1), any choice that doesnt have F or K is out (based on rule #2), and any choice that separates G and L is out (based on rule #3). Lo and behold, e) cannot be true because it does not contain either F or G as a driver. e) This is the correct answer. 3. Type of Question: LocalCould Be True According to rule #3, G and L must travel together, so we know these two will be in the same car. Since rules #1 and #2 tell us that H and J cant have L as their driver, they will go in the other carF will drive this car as she is the only common denominator between the two. K is the only person who hasnt been placed, and he could sit in either car. 1: L*, G 2: F*, J, H a) This is the correct answer. This could be true, since K could go in either car. 4. Type of Question: LocalMust Be True In this question were told that there is a 3-3 split, and that F is not one of the drivers. This means that G and K have to drive, and that J and H will be in separate cars in order to satisfy rules #1 and #2. L, as per rule #3, must travel with G. 1: K*, F, J 2: G*, L, H a) b) c) d) e) This cannot be true. This cannot be true. This is the correct answer. K must be the driver, so this cannot be true. This cannot be true.

Ivy Global

Preptests 61 Answers and Explanations (By Ivy Global)

5. Type of Question: GlobalCannot Be True This question should be approached with reference to rules, past work, and hypotheticals if necessary. a) This is possible, and we saw it in question #3. b) This is possible: 1: F*, H 2: K*, J, G, L c) This is possible: 1: K*, J 2: F*, H, G, L d) This is the correct answer. If K is alone with another driver, that driver would have to be G (so that the second car can satisfy rules #1 and 2 with F as the common driver). And, as we know, G must occur with L. This is incoherent. e) This is possible, and is the same as what we saw in question #3, with the drivers switched. Questions 6 11 The rules give us two ways this game can play out:

or In the first case, P or F can come first, and T or H could be last. In the second case, F or N could come first, and T or P could be last. 6. Type of Question: GlobalList This question should be approached with a rule sweep a) b) c) d) e) This is the correct answer. Eliminated by rule #2: N and J are older than T. Eliminated by rule #3: P is older than H and N, or H and N are older than P. Eliminated by rule #1: F is older than J and H. Eliminated by rule #2: N and J are older than T.

7. Type of Question: GlobalCould Be True Looking at both diagrams, we can see that only P, F, or N could come first. c) This is the correct answer. 8. Type of Question: GlobalCannot Be True F is the only artefact that cannot be fourth. In option 1, it requires J, T, and H to come after it, putting it in 3 and the earliest. In option 2, only N can come before F, so the latest it can come in 2.

Ivy Global

Preptests 61 Answers and Explanations (By Ivy Global)

a) This is the correct answer. 9. Type of Question: LocalMust Be True F could only be in 3 in option 1and in order for it to play out, the first two spaces must be P and N. P N F J/H J/H T c) This is the correct answer. 10. Type of Question: LocalCould Be True If P is in 1, then option2 is activatedso F or N can be in 2. b) This is the correct answer. 11. Type of Question: GlobalSubstitution We need to find a rule that has the same effect as N & J both being older than T on both options. a) b) c) d) T does not have to be older than H in either option. This is true, but it doesnt force J into a position that is older than T. A biconditional statement wont help us herewe need an absolute one. This is the correct answer. It makes sure that J and N are older than T, and the other rules keep F in its rightful place. e) Again, a biconditional statement wont help us secure an absolute rule.

Questions 12 17 This is game combines aspects of selection and sequencing, which makes it a bit more difficult. It may be helpful to do a quick run through of what the rules really say (and how you should write them out), before we move onto the game. If Q is selected Q-T (If we dont have a Q-T blockQ has not been selected (R, U, T, S have) S only in 1 or 3 If U is not selected (R, Q, T, S have)R in 2 (If U is selectedR is not in 2) The last two rules combined to make a biconditional statement. This means we can read each statement forward and backward (each statement is necessary and sufficient for the other.) 12. Type of Question: GlobalList This question should be approached with a rule sweep. a) b) c) d) e) Eliminated by rule #4: if R is in 2, then U is not selected. Eliminated by rule #2: S cannot be in 2. Eliminated by rule #1: If Q is selected, then Q-T. This is the correct answer. Eliminated by rule #3: if U is not selected, then R is in 2.

Ivy Global

Preptests 61 Answers and Explanations (By Ivy Global)


13. Type of Question: GlobalMust Be True Q requires T. Since only one of the runners are going to be out, and eliminating effectively eliminates two runners (both Q and T), we cant eliminate T. d) This is the correct answer. 14. Type of Question: Global: Fully Determined For this question, we have to remember that the correct answer fully determines the set-up. If were stuck, its likely not the right answer, so move quickly through. a) This tells us that U is also in the race, but not much else. Eliminate. b) This is the correct answer. We know that U is out here, and because we have the Q-T block, we can fill in the full diagram: S R Q T c) This tells us that U is also in the race, but not much else. Eliminate. d) This tells us that U is also in the race, but not much else. Eliminate. e) This tells us that the runners are U, Q, S, T, but nothing about the set-up. Eliminate. 15. Type of Question: GlobalCannot Be True This question should be approached with reference to the rules and to past work. a) This is the correct answer. Rule 2 would mean that S is in 3, so R is in 2. We cant have U, then, due to rule #3 and 4but we also cant split up Q and T. b) This works, and a hypothetical proves it possible: S Q T U c) This works, and a hypothetical proves it possible: S T R U d) This works, and a hypothetical proves it possible: Q T R U e) This works, and a hypothetical proves it possible: R S U T 16. Type of Question: LocalMust Be True If U is selected for 1, then rule #4 tells us that R cannot be in 2. As per rule #2, we also know that S cant be in 2, so were left with only Q or T for that spot. A quick set-up shows us there are only two ways this game could play out: U T S R or U Q T R a) b) c) d) e) This could be true, but doesnt have to be. This could be true, but doesnt have to be. This could be true, but doesnt have to be. This could be true, but doesnt have to be. This is the correct answer.

17. Type of Question: LocalCould Be True Since S can only be in slots 1 or 3, there are two ways this game could play out: Q T S R or S R/U Q T Only Q or S could run the first race. b) This is the correct answer.

Ivy Global

Preptests 61 Answers and Explanations (By Ivy Global)


Questions 18 23 This is a fairly straightforward linear sequencing game. Make sure to keep track of the rules as you move the questions and it is manageable. 18. Type of Question: GlobalList This question should be approached with a rule sweep. a) b) c) d) e) Eliminated by rule #4: F must be later than L. Eliminated by rule #5: L cannot be on 2. Eliminated by rule #3: J must be after M. This is the correct answer. Eliminated by rule #5: L cannot be on 2.

19. Type of Question: GlobalCannot Be True This question should be left for the end so we can approach it using rules, past work, and hypotheticals when necessary. a) We saw J could go in 2 in question 22. b) We saw J in 3 in question 20. c) This is the correct answer. It doesnt leave room for the M/H _ _ M/H block without forcing L into 2. d) J could go in 6: L F M G K J H e) We saw J in 7 in question 21. 20. Type of Question: LocalCould Be True According to rule #3, M must come before J, so M will have to fall into 1 or 2. Were left with H, and out giant chain L-F-GK. The H must come after J in order to fit with rule #1, which means that L will have to come before J in order for the chain to fit. Since rule #5 stops us from putting L in 2, we know a part of the set-up for sure: L M J F _ _ _ a) b) c) d) e) This is not possibleit would force L into 2. This is not possibleG needs to come immediately before K. This is not possibleit would separate the GK block. This is the correct answer: L M J F G K H This is not possible.

21. Type of Question: LocalCould Be True This question adds to our giant chain: L-F-GK-M-J. Only H needs to be added in, and it can come only between L and F or between F and G in order to satisfy rule #1. Only H or F can fall into spot 3. a) b) c) d) e) This would force H to break up the GK block to satisfy rule #1. This is the correct answer. J must be last, since it comes after M. L must be firstthere arent enough entities that can come before it to push it to 3. M must be 6there arent enough entities that can come after it to push it up to 3.

Ivy Global

Preptests 61 Answers and Explanations (By Ivy Global)

22. Type of Question: LocalMust Be True We immediately know that if G is in 5, K is in 6, due to rule #2. L and F must come before GK as per rule #4. Since M has to come before J, it cant be in the last spot. If we tried to put J into 7, wed run into the problem where we have to force L into 2 (M/H _ _ M/H G K J), so that cant be trueand only H can go in 7. _ _ _ _ G K H a) b) c) d) e) F could fall in 2, 3, or 4. This is the correct answer. J could fall into 2, 3, or 4. L could fall into 1 or 3. M could fall into 1, 2, 3, or 4.

23. Type of Question: GlobalCould Be True The L-F-GK block limits L to being placed into 1, 2, 3, or 4. 2 is out, according to rule #5. If we put L in 4, and L F G K fills up the last four spots, then we dont have room for out M/H _ _ M/H blockso L can only fall in 1 or 3. a) This is the correct answer.

Ivy Global

You might also like